静雅斋成就

更优秀的你

总体估计中的相关公式 | 高一使用

前言

直接针对字母进行思维,其要求比针对数字思维的要求更高一些,是现代学生需要具备的一项技能;需要好好练习;

相关公式

【人教 2019 A 版 \(P_{215}\) 练习 2】 数据 \(x_1\)\(x_2\)\(\cdots\)\(x_n\) 的方差为 \(s_x^2\), 数据 \(y_1\)\(y_2\)\(\cdots\)\(y_n\) 的方差为 \(s_y^2\)\(a\)\(b\) 为常数. 证明:

(1) . 如果 \(y_1=x_1+b\)\(y_2=x_2+b\)\(\cdots\)\(y_n=x_n+b\), 那么 \(s_y^2\)\(=\)\(s_x^2\);

证明:由于 \(s_x^2=\cfrac{1}{n}\sum\limits_{i=1}^{n}(x_i-\bar{x})^2\), 且 \(\bar{y}=\cfrac{1}{n}\sum\limits_{i=1}^{n}(x_i+b)=\cfrac{1}{n}\sum\limits_{i=1}^{n}x_i+\cfrac{1}{n}\sum\limits_{i=1}^{n}b=\bar{x}+b\)

\(s_y^2=\cfrac{1}{n}\sum\limits_{i=1}^{n}(y_i-\bar{y})^2=\cfrac{1}{n}\sum\limits_{i=1}^{n}[(x_i+b)-(\bar{x}+b)]^2=\cfrac{1}{n}\sum\limits_{i=1}^{n}(x_i-\bar{x})^2=s_x^2\)

故有 \(s_y^2\)\(=\)\(s_x^2\);

解后反思:将这组数据按照 \((i,x_i)\) 的格式绘制在坐标系中,从形上来理解,给一组原始数据统一加某个确定值,这组数据会沿着 \(y\) 轴上下统一移动,不会改变平均数,不会改变原始数据的集中或离散程度,也就是方差不会变化,故 \(s_y^2\)\(=\)\(s_x^2\)

(2) . 如果 \(y_1=ax_1\)\(y_2=ax_2\)\(\cdots\)\(y_n=a x_n\), 那么 \(s_y^2\)\(=\)\(a^2\)\(\cdot s_x^2\).

证明:由于 \(s_x^2=\cfrac{1}{n}\sum\limits_{i=1}^{n}(x_i-\bar{x})^2\), 且 \(\bar{y}=a\cdot\bar{x}\)

\(s_y^2=\cfrac{1}{n}\sum\limits_{i=1}^{n}(y_i-\bar{y})^2\)\(=\)\(\cfrac{1}{n}\sum\limits_{i=1}^{n}(a\cdot x_i-a\cdot\bar{x})^2\)

\(=\)\(\cfrac{1}{n}\sum\limits_{i=1}^{n}a^2(x_i-\bar{x})^2\)\(=\)\(a^2\cdot\cfrac{1}{n}\sum\limits_{i=1}^{n}(x_i-\bar{x})^2\)\(=\)\(a^2\cdot s_x^2\)

解后反思:将这组数据按照 \((i,x_i)\) 的格式绘制在坐标系中,从形上来理解,给一组原始数据统一乘以某个确定值,相当于将这组数据沿着 \(y\) 轴方向压缩或拉伸,会改变平均数,故一般会改变原始数据的集中或离散程度,故 \(s_y^2\)\(=\)\(a^2\)\(\cdot s_x^2\).

【人教 2019 A 版 \(P_{216}\) 习题9.2的第 4 题】数据 \(x_1\)\(x_2\)\(\cdots\)\(x_n\) 的方差和标准差分别为 \(s_x^2\)\(s_x\), 数据 \(y_1\)\(y_2\)\(\cdots\)\(y_n\) 的方差和标准差分别为 \(s_y^2\)\(s_y\). 若 \(y_1=a x_1+b\)\(y_2=a x_2+b\)\(\cdots\)\(y_n=a x_n+b\) 成立, \(a\)\(b\) 为常数, 证明: \(s_y^2=\)\(a^2\cdot\)\(s_x^2\)\(s_y\)\(=\)\(|a|\)\(s_x\).

证明: 由于 \(\bar{y}=\cfrac{1}{n}\sum\limits_{i=1}^{n}(ax_i+b)=\cfrac{1}{n}\sum\limits_{i=1}^{n}ax_i+\cfrac{1}{n}\sum\limits_{i=1}^{n}b=a\cdot\cfrac{1}{n}\sum\limits_{i=1}^{n}x_i+b=a\bar{x}+b\)

由于 \(s_x^2=\cfrac{1}{n}\sum\limits_{i=1}^{n}(x_i-\bar{x})^2\)

\(s_y^2=\cfrac{1}{n}\sum\limits_{i=1}^{n}(y_i-\bar{y})^2\)\(=\)\(\cfrac{1}{n}\sum\limits_{i=1}^{n}[(a\cdot x_i+b)-(a\cdot\bar{x}+b)]^2\)

\(=\)\(\cfrac{1}{n}\sum\limits_{i=1}^{n}[a\cdot(x_i-\bar{x})]^2\)\(=\)\(\cfrac{1}{n}\sum\limits_{i=1}^{n}a^2\cdot(x_i-\bar{x})^2\)\(=a^2\cfrac{1}{n}\sum\limits_{i=1}^{n}(x_i-\bar{x})^2=a^2s_x^2\)

所以,\(s_y^2=a^2s_x^2\),故 \(s_y=|a|s_x\)

解后反思:将这组数据按照 \((i,x_i)\) 的格式绘制在坐标系中,从形上来理解,给一组原始数据统一乘以某个确定值,然后统一加某个确定值, 相当于将这组数据沿着 \(y\) 轴方向压缩或拉伸,再将这组数据沿着 \(y\) 轴上下统一移动,故会改变平均数,一般会改变原始数据的集中或离散程度,故 \(s_y^2\)\(=\)\(a^2\)\(\cdot s_x^2\).

【2024高一数学训练题,性质应用】 某班有 \(40\) 名学生,在某次考试中,全班的平均分为 \(70\) 分,最高分为 \(100\) 分,最低分为 \(50\) 分,现将全班每个学生的分数以 \(y_i=ax_i+b\) (其中 \(a>0\) ) 进行调整,其中 \(x_i\) 是第 \({i}\) 个学生的原始分数,\(y_i\) 是第 \({i}\) 个学生的调整后的分数,调整后,全班最高分为 \(100\) 分,最低分为 \(60\) 分, 则 \(\qquad\)

$A$.$\textbf{调整后分数的极差和原始分数的极差相同}$
$B.$$\textbf{调整后分数的中位数要高于原始分数的中位数}$
$C.$$\textbf{调整后分数的标准差和原始分数的标准差相同}$
$D.$$\textbf{调整后分数的众数个数要多于原始分数的众数个数}$

解:将数据 \((50,60)\)\((100,100)\) 代入 \(y_i=ax_i+b\),解得 \(a=0.8\)\(b=20\),即 调整公式为 \(y_{i}=0.8x_{i}+20\)

原始数据的极差为 \(100-50=50\),调整后的数据的极差为 \(100-60=40\),故选项 \(A\) 错误;

设原始数据的中位数为 \(a(50<a<100)\),则调整后的中位数为 \(0.8a+20\)\((0.8a+20)-a=-0.2a+20>0\),故选项 \(B\) 正确;

原始数据的标准差设为 \(s\),则根据性质得到,调整后的数据的标准差为 \(0.8s\),故选项 \(C\) 错误;

由公式 \(y_{i}=0.8x_{i}+20\)可知,将原始数据统一缩小 \(0.8\) 倍,再统一向上平移 \(20\) 个单位,则原始数据的众数和调整后的数据的众数的数值可能会不一样,但两组数据的众数的个数一定是相同的,故选项 \(D\) 错误;综上所述,选 \(B\) .

【人教 2019 A 版 \(P_{216}\) 习题9.2的第 5 题】 数据 \(x_1\)\(x_2\)\(\cdots\)\(x_n\) 的方差 \(s^2=0\), 证明: 所有的 \(x_i\)(\(i\)\(=\)\(1\)\(2\)\(\cdots\)\(n\)) 都相同.

证明:由于 \(s_x^2=\cfrac{1}{n}\sum\limits_{i=1}^{n}(x_i-\bar{x})^2=0\),故 \(x_i-\bar{x}=0\),即 \(x_i=\bar{x}\)\(x_i\)(\(i\)\(=\)\(1\)\(2\)\(\cdots\)\(n\)) ,得证 .

【人教 2019 A 版 \(P_{218}\) 习题9.2的第 11 题】已知总体划分为 \(3\) 层,通过分层随机抽样, 各层抽取的样本量、样本平均数和样本方差分别为: \(l\)\(\bar{x}\)\(s_1^2\)\(m\)\(\bar{y}\)\(s_2^2\)\(n\)\(\bar{z}\)\(s_3^2\) . 记总的样本平均数为 \(\bar{w}\), 样本方差为 \(s^2\), 证明:

(1). \(\bar{w}\)\(=\)\(\cfrac{l}{l+m+n}\cdot\bar{x}\)\(+\)\(\cfrac{m}{l+m+n}\cdot\bar{y}\)\(+\)\(\cfrac{n}{l+m+n}\cdot\bar{z}\);

证明:由于 \((l+m+n)\bar{w}=l\cdot\bar{x}+m\cdot\bar{y}+n\cdot\bar{z}\)

\(\bar{w}\)\(=\)\(\cfrac{l}{l+m+n}\cdot\bar{x}\)\(+\)\(\cfrac{m}{l+m+n}\cdot\bar{y}\)\(+\)\(\cfrac{n}{l+m+n}\cdot\bar{z}\); [1]

(2). \(s^2\)\(=\)\(\cfrac{1}{l+m+n}\)\(\bigg\{l\cdot\left[s_1^2+(\bar{x}-\bar{w})^2\right]\)\(+\)\(m\cdot\left[s_2^2+(\bar{y}-\bar{w})^2\right]\)\(+\)\(n\cdot\left[s_3^2+(\bar{z}-\bar{w})^2\right]\bigg\}\).

待有空整理;

  • 平均数、方差、标准差的性质推广

如果一组样本数据\(x_1\)\(x_2\)\(\cdots\)\(x_n\),其平均数为\(\bar{x}\),方差为\(s^2\),标准差为\(s\)

则样本数据\(ax_1+b\)\(ax_2+b\)\(\cdots\)\(ax_n+b\),其平均数为\(a\bar{x}+b\),方差为\(a^2\cdot s^2\),标准差为\(|a|\cdot s\)

【2021年高三文数三轮模拟题】若样本 \(x_1+1\)\(x_2+1\)\(x_3+1\)\(\cdots\)\(x_n+1\) 的平均数为 \(10\),方差为 \(2\),则对于样本 \(2x_1+3\)\(2x_2+3\)\(2x_3+3\)\(\cdots\)\(2x_n+3\) ,下列结论正确的是【\(\quad\)

$A.$平均数为 $20$,方差为 $4$
$B.$平均数为 $11$,方差为 $4$
$C.$平均数为 $21$,方差为 $8$
$D.$平均数为 $20$,方差为 $8$

解析:由于样本 \(x_1+1\)\(x_2+1\)\(x_3+1\)\(\cdots\)\(x_n+1\) 的平均数为 \(10\)

则样本 \(x_1\)\(x_2\)\(x_3\)\(\cdots\)\(x_n\) 的平均数为 \(9\)[2]

对于样本 \(2x_1+3\)\(2x_2+3\)\(2x_3+3\)\(\cdots\)\(2x_n+3\)

其平均数为 \(2\times 9+3=21\),方差为 \(2^2\times 2=8\) ,故选 \(C\).


  1. 相关公式,【人教 2019 A 版 \(P_{184}\) 练习1】 数据 \(x_1\)\(x_2\)\(\cdots\)\(x_m\) 的平均数是 \(\bar{x}\),数据 \(y_1\)\(y_2\)\(\cdots\)\(y_n\) 的平均数是 \(\bar{y}\),则 \(\cfrac{\sum\limits_{i=1}^{m}x_i+\sum\limits_{i=1}^{n}y_i}{m+n}=\cfrac{m}{m+n}\bar{x}+\cfrac{n}{m+n}\bar{y}\) ↩︎

  2. 解释:由于 \(\cfrac{(x_1+1)+(x_2+1)+\cdots+(x_n+1)}{n}=10\),则 \(\cfrac{(x_1+x_2+\cdots+x_n)+n}{n}=10\)
    \(x_1+x_2+\cdots+x_n=9n\),故 \(\cfrac{x_1+x_2+\cdots+x_n}{n}=9\)↩︎

posted @ 2024-06-18 08:14  静雅斋数学  阅读(16)  评论(0编辑  收藏  举报
您已经努力一段时间了
活动活动喝杯咖啡吧
                  ----静雅斋